Unterhaltung
Menschen Wissenschaft Politik Mystery Kriminalfälle Spiritualität Verschwörungen Technologie Ufologie Natur Umfragen Unterhaltung
weitere Rubriken
PhilosophieTräumeOrteEsoterikLiteraturAstronomieHelpdeskGruppenGamingFilmeMusikClashVerbesserungenAllmysteryEnglish
Diskussions-Übersichten
BesuchtTeilgenommenAlleNeueGeschlossenLesenswertSchlüsselwörter
Schiebe oft benutzte Tabs in die Navigationsleiste (zurücksetzen).

Perpetuum Mobile (mit Skizze)

334 Beiträge ▪ Schlüsselwörter: Perpetuum Mobile, Unendlich Energie ▪ Abonnieren: Feed E-Mail

Perpetuum Mobile (mit Skizze)

30.04.2012 um 05:30
@OpenEyes
Zitat von OpenEyesOpenEyes schrieb:daher wird an dieser Kugel in Bewegungsrichtung nur die Druckdifferenz zwischen Tank und Rohr wirksam.
natürlich wird sie wirksam. Bleibt aber während des Weges p1-p2 konstant....

gibs dafür auch ne formel?

Anzeige
1x zitiertmelden

Perpetuum Mobile (mit Skizze)

30.04.2012 um 05:30
@Primpfmümpf

Nachsatz: Wenn Du das Ding baust und es läuft, darf natürlich die Wassersäule im Rohr nicht absinken, sonst hast Du nur ein unnötig kompliziertes Wasserrad.


melden

Perpetuum Mobile (mit Skizze)

30.04.2012 um 05:32
Was hätte die senkung im rohr mit einem Wasserrad zu tun?

achso verstehe. ja das wäre eine verschwenderische konstruktion


...

ne ich raf das doch nicht was du damit meinst..


1x zitiertmelden

Perpetuum Mobile (mit Skizze)

30.04.2012 um 05:34
@Primpfmümpf
Zitat von PrimpfmümpfPrimpfmümpf schrieb:natürlich wird sie wirksam. Bleibt aber während des Weges p1-p2 konstant....

gibs dafür auch ne formel?
Nein, sie ist zwar vorhanden, der Vektor steht aber normal zur Bewegungsrichtung und hat daher keine Auswirkung auf F x S (Kraft mal Weg)
Zitat von PrimpfmümpfPrimpfmümpf schrieb:Was hätte die senkung im rohr mit einem Wasserrad zu tun?
Wenn der Wasserstand im Rohr sinkt, dann kannst Du Energie gewinnen - genau die potenzielle Energie des absinkenden Wassers. Und du musst Wasser nachfüllen, wofür Du wieder genau so viel Energie aufwenden musst wie Du entnommen hast.


melden

Perpetuum Mobile (mit Skizze)

30.04.2012 um 05:37
@OpenEyes

Das sind wielleicht mal n paar tropfen am Tag wenn nichts rumspritzt. Besser als für solch einen Energiegewinn 100 ml benzien zu tanken


1x zitiertmelden

Perpetuum Mobile (mit Skizze)

30.04.2012 um 05:41
@Primpfmümpf
Zitat von PrimpfmümpfPrimpfmümpf schrieb:Das sind wielleicht mal n paar tropfen am Tag wenn nichts rumspritzt. Besser als für solch einen Energiegewinn 100 ml benzien zu tanken
Du hast mich anscheinend nicht verstanden:

Du kannst nur die Energie entnehmen, welche sich aus dem Absinken des Wasserspiegels ergibt. Und wenn nur ein paar Tropfen am Tag weniger im Rohr sind, kannst Du auch nur so viel Energie entnehmen wie dem Fall der Tropfen von der Oberfläche im Rohr zur Oberfläche im Tank entspricht. Nichts mit 100ml Benzin, bestenfalls 1/100 Tropfen


melden

Perpetuum Mobile (mit Skizze)

30.04.2012 um 05:44
@OpenEyes

Warum sollte das Wasser sinken. Bei diesem Experiment wäre es nur vom Vorteil das wasser immer schön an der säulenkante dippen zu lassen. Ich verstehe das jetzt nicht.

Du weist dass es mir nur um Luftbälle an einer Zahnkette über zahnräder geht?

Wenn der pegel in der säule sinken würde, hätten die obersten Bälle keinen Auftrieb mehr


melden

Perpetuum Mobile (mit Skizze)

30.04.2012 um 05:47
momnet mal.

hm die Zahnkette müste auch stoßend wirken um die Bälle in den Tank nach unten zu drücken.. vielleicht hast du doch recht.

...

ne doch nicht.. :)


1x zitiertmelden

Perpetuum Mobile (mit Skizze)

30.04.2012 um 05:59
Zitat von OpenEyesOpenEyes schrieb:Der Druck in einer Flüssigkeit hängt ausschließlich von der Höhe und dem spezifischen Gewicht der Flüssigkeit ab.
Links von der untersten Kugel entspricht er der Tiefe des Tanks, rechts der Höhe der Flüssigkeit im Rohr. Die Differenz entspricht genau der Höhe der Flüssigkeit im Rohr oberhalb der Oberfläche im Tank.

Gegen diese Druckdifferenz muss die unterste Kugel (die vom Tank ins Rohr wandern soll) bewegt werden. Dafür muss Energie aufgewendet werden. Und diese Energie ist genau gleich der Energie, welche Du durch die Auftriebskraft der Kugeln im Rohr auf dem gleichen Weg gewinnen kannst
irgendwie glaub ich nun doch dass du wieder recht hast.

Das heist der Tischtennisball am meeresgrund steigt zwar genauso schnell auf wie in der 10 meter langen säule, aber der aus dem Meer hat mehr kraft..


melden

Perpetuum Mobile (mit Skizze)

30.04.2012 um 06:08
@Primpfmümpf
Zitat von PrimpfmümpfPrimpfmümpf schrieb:hm die Zahnkette müste auch stoßend wirken um die Bälle in den Tank nach unten zu drücken.. vielleicht hast du doch recht.

...

ne doch nicht..
Nein, das ist nicht das Problem.

Machen wir die Rechnung, dann wird es vielleicht klar.

1) Der Druck einer Wassersäule von 10 Metern Höhe beträgt 1 Kp/cm²
2) Unsere Schwimmkörper sollen 1 dm³ (1 Liter) Volumen haben, dann hat ein Schwimmkörper einen Auftrieb von 1 Kp.
3) Der Schwimmkörper im Übergangskanal muss dicht abschließen, sonst fließt das Wasser aus dem Rohr sofort in den Tank (kommunizierende Gefäße stellen immer gleiche Oberflächen her - siehe Schlauchwaage)
4) Auf den Schwimmkörper im Rohr wirkt also auf der Rohr - Seite ein Druck von 1 Kp/cm². Der Querschnitt eines kugelförmigen Schwimmkörpers ist rund ein dm², also 100 cm², daher Druck auf der Rohr - Seite rund 100 Kp
5) Auf der Tank - Seite haben wir einen Druck von 0,1 Kp/cm², Gesamtdruck also rund 10 Kp
6) Druckdifferenz, gegen die Du den Schwimmkörper im Rohr bewegen musst also rund 90 Kp
7) Da die Kugeln rund 10 cm Durchmesser haben, kannst Du im Rohr oberhalb der Tank - Oberfläche maximal 90 Kugeln unterbringen, gesamt - Auftrieb 90 Kp
8) Da die Kugeln an einer Kette hängen, muss der Weg der aufschwimmenden Kugeln gleich dem Weg der Kugel durch den Kanal sein.
9) Die Kraft, die auf die Kugel im Kanal wirkt beträgt 90 Kp, die Auftriebskraft der Kugeln im Rohr ebenfalls 90 Kp
10) Du gewinnst also [Länge des Kanals] x 90 Kp Energie durch den Auftrieb. Davon musst Du [Länge des Kanals] x 90 Kp aufwenden, um die Kugel durch den Kanal zu ziehen.

Ergebnis: 90 Kp x Weglänge - 90 Kp x Weglänge = 0
q.e.d.


1x zitiertmelden

Perpetuum Mobile (mit Skizze)

30.04.2012 um 06:12
@Primpfmümpf

Nachsatz: Dein Denkfehler liegt darin, dass Du Druck und Auftrieb in einen Topf wirfst.

Der Auftrieb hängt nur vom Volumen ab, der Druck aber von der Tiefe.

Und auf die Kugel im Kanal wirken Drücke, auf die im Rohr aber nur Auftriebskräfte.


melden

Perpetuum Mobile (mit Skizze)

30.04.2012 um 06:19
@OpenEyes
Zitat von OpenEyesOpenEyes schrieb:Der Schwimmkörper im Übergangskanal muss dicht abschließen, sonst fließt das Wasser aus dem Rohr sofort in den Tank
Kannst du mir sagen warum kein Wasser vom Rohr in den Tank darf? Der pegel bleibt doch im Tank und im rohr immer gleich..


1x zitiertmelden

Perpetuum Mobile (mit Skizze)

30.04.2012 um 06:43
@Primpfmümpf
Zitat von PrimpfmümpfPrimpfmümpf schrieb:Kannst du mir sagen warum kein Wasser vom Rohr in den Tank darf? Der pegel bleibt doch im Tank und im rohr immer gleich..
Wenn Wasser vom Rohr in den Tank fließt, dann steigt der Pegel im Tank und er Pegel im Rohr sinkt ab. Nach kurzer Zeit hast Du in beiden den gleichen Pegel und vorbei ist die Freude. Du müsstest Wasser aus dem Tank in das Rohr nachfüllen und dafür die gleiche Energie aufwenden, die Du vorher entnehmen konntest.

Deine Anordnung wird im Übrigen ohnehin von selbst zu laufen anfangen, aber nicht in die Richtung, in die Du meinst, sondern genau umgekehrt. Und sie wird ein paar Sekunden laufen, so lange, bis der Pegel im Rohr gleich dem Pegel im Tank ist. (Der Auftrieb der Kugeln kann höchstens so groß sein, wie diei Druckdifferenz an der Kugel im Kanal, was in der Praxis aber nicht erreichbar ist, da die Kugeln dazu unmittelbar aneinander liegen müssten und die Kette dann steif wäre.

Deine Anordnung hat nämlich noch ein paar zusätzliche Probleme, die ich der Einfachheit halber nicht erwähnt habe:

1) damit das Ganz ein PM ist, darf der Wasserspiegel im Rohr nicht sinken - Nachfüllen würde die Energie, die Du vorher entnommen hast zur Gänze aufbrauchen
2) Um das zu gewährleisten, muss immer mindestens eine Kugel im Kanal sein, der Abstand der Kugeln muss also kleiner sein, als die Länge des Kanals, damit der Kanal immer geschlossen bleibt
3) In diesem Fall wirkt das Ganze, wenn Du es von außen zum Beispiel durch eine Kurbel in der von Dir vermuteten Richtung in Bewegung setzt, als Pumpe und pumpt mit Hilfe Deiner Muskelkraft Wasser aus dem Tank in das Rohr, da die nächste Kugel in den Kanal eintreten muss, ehe die vorherige ihn verlässt. Zwischen den Kugeln ist also immer eine gewisse Menge Wasser eingeschlossen, welches von den Kugeln in das Rohr transportiert wird.


1x zitiertmelden

Perpetuum Mobile (mit Skizze)

30.04.2012 um 06:47
@Primpfmümpf

Nachsatz: Wenn Du das Rohr immer wieder aus einem Fluss nachfüllst, dann hast Du eben kein PM, sondern ein kompliziertes Wasserrad und Du nimmst die Energie aus der Sonne, welche dafür sorgt, dass Wasser im Meer verdunstet und in Form von Regen den Fluss speist.


melden

Perpetuum Mobile (mit Skizze)

30.04.2012 um 06:55
@OpenEyes

Ich hoffe du gehst nicht davon aus dass das Wasser in meinem Versuch fließt wie hir bei 0:27 http://www.youtube.com/watch?feature=player_embedded&v=287qd4uI7-E und die kugeln dann befördert.

Das Wasser bleibt still.
Zitat von OpenEyesOpenEyes schrieb:Deine Anordnung wird im Übrigen ohnehin von selbst zu laufen anfangen, aber nicht in die Richtung, in die Du meinst, sondern genau umgekehrt
Warum?
Ich meine das Auftribskapital ist in der Säule(also Rohr oder :) ?) größer als die wenigeren kugeln im Tank.
Zitat von OpenEyesOpenEyes schrieb:Und sie wird ein paar Sekunden laufen, so lange, bis der Pegel im Rohr gleich dem Pegel im Tank ist
oder denkst du hir dass im Falle des Uhrzeigersinnverlaufs die kugeln durch den Sog das Wasser aus dem rohr in den Tang ziehen...


1x zitiertmelden

Perpetuum Mobile (mit Skizze)

30.04.2012 um 07:09
@Primpfmümpf
Zitat von PrimpfmümpfPrimpfmümpf schrieb:Ich hoffe du gehst nicht davon aus dass das Wasser in meinem Versuch fließt wie hir bei 0:27 http://www.youtube.com/watch?feature=player_embedded&v=287qd4uI7-E und die kugeln dann befördert.

Das Wasser bleibt still.
Das PM im Video ist ein netter Gag, aber ohne die Pumpe im Schlauch läuft es nicht :)
Zitat von PrimpfmümpfPrimpfmümpf schrieb:Warum?
Ich meine das Auftribskapital ist in der Säule(also Rohr oder :) ?) größer als die wenigeren kugeln im Tank.
Nein, es ist immer kleiner, da die Kraft, welche auf die Kugel im Kanal wirkt, kein Auftrieb sondern eine Druckdifferenz ist. Dei Kugeln unterhalb des Pegels im Kanal kannst Du vernachlässigen, ihr Auftrieb hebt sich gegenseitig auf.
Zitat von PrimpfmümpfPrimpfmümpf schrieb:oder denkst du hir dass im Falle des Uhrzeigersinnverlaufs die kugeln durch den Sog das Wasser aus dem rohr in den Tang ziehen...
Nicht durch den Sog, sondern dadurch, dass immer eine Kugel im Kanal sein muss um ihn dicht zu halten. Das heißt, es ist immer eine Wassersäule zwischen zwei Kugeln im Kanal eingeschlossen und diese Wassermenge wird mit den Kugeln vom Rohr in den Tank transpotriert.


2x zitiertmelden

Perpetuum Mobile (mit Skizze)

30.04.2012 um 07:25
@OpenEyes

Du hast immernoch nicht gesagt warum du davon ausgehst dass der pegel bei tank und säule nicht wie von mir erdacht stehts gleich bleibt, auch wenn die kugelkette laufen würde.
Zitat von OpenEyesOpenEyes schrieb:Nein, es ist immer kleiner, da die Kraft, welche auf die Kugel im Kanal wirkt, kein Auftrieb sondern eine Druckdifferenz ist.
(Kanal = P1 -> P2 oder? (nur um missverständnisse zu vermeiden)

Stell dir vor der Kanal ist 20 cm lang und die säule 10 Meter hoch. Der Abstand der Bälle beträgt 10 cm bei 10 cm durchmesser.

Da läuft solch nur eine Kugel schon aleine durch den Auftrieb mit druck gegen die obere kanalwand.
Schleift aber nicht wegen dem Zahnrad. Die kugel im Kanal kann außer acht gelassen werden.

Denn wenn der Tank mal übertriebener Weise gesagt nur 10cm tief wäre und der kanal 100 Meter lang, dann würde zwei Kugeln in der Säule(eine noch im Tank) bereits eine Zugkraft richtung säule (gegen uhrzeigersinn) auslösen.
Nun ist der Tank aber tiefer und mehr Wasser drückt im kanal.
Und deshalb ist auch wieder die Säule mit mehr wasser gefüllt weil es vom Tank nach drückt, aber der Kanal mit dem nun höheren druck ist irelevand


melden

Perpetuum Mobile (mit Skizze)

30.04.2012 um 07:41
Zitat von OpenEyesOpenEyes schrieb:Das heißt, es ist immer eine Wassersäule zwischen zwei Kugeln im Kanal eingeschlossen und diese Wassermenge wird mit den Kugeln vom Rohr in den Tank transpotriert
durch den Druck von Tank und Säule im Kanal fließt das Wasser zwischen den Kugeln immer zurück zum Druckschwerpunkt -> dem Kanal


melden

Perpetuum Mobile (mit Skizze)

30.04.2012 um 09:21
Das archimedische Prinzip lautet:
Die Auftriebskraft eines Körpers in einem Medium ist genauso groß wie die Gewichtskraft des vom Körper verdrängten Mediums.
Steigen, Sinken, Schweben...
Das archimedische Prinzip gilt nur genau dann streng, wenn das verdrängte Medium inkompressibel (nicht zusammendrückbar) ist.

ich denke es würde funktionieren - wir hatten schon mal an anderer Stelle darüber gesprochen @OpenEyes (;
Freie Energie/Perpetuum Mobile (Seite 59) (Beitrag von 1.21Gigawatt) (geht über einige Seiten)
... es bliebe am Ende jedoch immer nur ein "umgekehrtes Wasserrad" .

wenn man sich darüber Gedanken macht lernt man wenigsten etwas über Auftrieb und Druck!
Wikipedia: Hydrostatisches Paradox
Wikipedia: Archimedisches Prinzip
Wikipedia: Auftriebskraft
Wikipedia: Hydrostatischer Druck


melden

Perpetuum Mobile (mit Skizze)

30.04.2012 um 10:09
edit
Youtube: Auftrieb in Flüssigkeiten
Auftrieb in Flüssigkeiten
Externer Inhalt
Durch das Abspielen werden Daten an Youtube übermittelt und ggf. Cookies gesetzt.



Anzeige

melden